Which one of the following pairs of employees is such that at least one member of the pair volunteers?

Julie-V on July 27, 2019

Tips for This Question Type

Hi LSAT Max, This question type tends to trip me up whenever I come across it during a game. Watching the video explanation helped me, but I was wondering if there were any general rules/tips you have when dealing with this question for in/out games in general. Is the key to focus on the binaries where at least one must be in? Thank you in advance for the help!

Replies
Create a free account to read and take part in forum discussions.

Already have an account? log in

Ravi on August 8, 2019

@Julie-V,

Great question.

Looking at the rules, we can combine them to make a chain:

/L - >R - >M - >T - >/F

/L - >R - >M - >T - >/V - >S

The question asks us which pair contains at least one member who must
be in. This question, and others like it, are asking us to find
'either/or' relationships, and these are often revealed in the
conditional chains that we make from the rules.

Looking at our big chain, we see that L and R are in an either/or
relationship (/L - >R). If L is out, then R must be in, and if R is
out, then L must be in. Likewise, L and M, L and T, L and S, and V and
S are in either/or relationships. The correct answer choice will be
one of these pairs.

Looking at the answer choices, we see (B) says that L and M are a pair
where at least one of them must be in. As seen in our chain, this is
true, so (B) is the correct answer choice.

Generally, this strategy will work really well for this type of
question when you encounter it in the future.

Does that make sense? Let us know if you have any other questions!

DrKumar on December 18, 2019

Thanks Ravi, this is a great way to figure out these types of questions... Much appreciated!